Grain

This topic has expert replies
Master | Next Rank: 500 Posts
Posts: 157
Joined: Sat Jan 30, 2016 5:55 am
Thanked: 1 times

Grain

by eitijan » Wed May 11, 2016 4:46 am
Source: GMATPrep

Experts, please comment.
OA E
Attachments
CR_GMATPrep.JPG

User avatar
Legendary Member
Posts: 2131
Joined: Mon Feb 03, 2014 9:26 am
Location: https://martymurraycoaching.com/
Thanked: 955 times
Followed by:140 members
GMAT Score:800

by MartyMurray » Tue May 17, 2016 3:31 pm
eitijan wrote:Gortland has long been narrowly self-sufficient in both grain and meat. However, as per capita income in Gortland has risen toward the world average, per capita consumption of meat has also risen toward the world average, and it takes several pounds of grain to produce one pound of meat. Therefore, since per capita income continues to rise, whereas domestic grain production will not increase, Gortland will soon have to import either grain or meat or both.

Which of the following is an assumption on which the argument depends?

(A) The total acreage devoted to grain production in Gortland will not decrease substantially.

(B) The population of Gortland has remained relatively constant during the country's years of growing prosperity.

(C) The per capita consumption of meat in Gortland is roughly the same across all income levels.

(D) In Gortland, neither meat nor grain is subject to government price controls.

(E) People in Gortland who increase their consumption of meat will not radically decrease their consumption of grain.
Conclusion: Gortland will have to import grain or meat or both.

Reasoning: As incomes go up, people in Gortland will consume more meat. Grain is used in meat production, and domestic grain production will not increase.

In this one there are multiple components to the argument. I don't even want to bother considering what the answer might be. So I am just going to go straight to the answer choices.

(A) I guess if you were not paying attention you might somehow twist this one to make it an assumption that the argument is based on, but really the argument implies that grain will not be sufficient. So it not assuming that land used for grain production will not decrease, as if the land used were to decreas, the probable result would be a decrease in grain production, an event that could actually contribute to the outcome the argument predicts.

(B) This is interesting, and could trap you if you let it, because if the population were to decrease, the argument might not work. However, if the population were to increase, the argument would work fine. So actually the argument is not based on an assumption that the population remain constant.

(C) This sounds nice, but really it's irrelevant. Whether the per capita consumption is the same across income levels or not, the per capita consumption of the entire population is what it is.

(D) Price controls could have various effects, but the argument is not assuming that there are no price controls. That statement is so general that it becomes irrelevant. What if there were some small price control that has little effect, for instance? The outcome would be unaffected. Also, how price controls would change the outcome is not clear. So don't make up a convoluted story and base choosing this answer choice on the story that you make up.

(E) Hmm. If people were to eat much less grain when they eat more meat, then the increase in grain needed for producing meat could be offset by a decrease in grain consumed by people. If that offseting were to occur, there would be sufficient grain in Gortland without importation. So the argument is based on the assumption that reduced grain consumption by people will not offset increased use of grain in meat production.

So the correct answer is E.
Marty Murray
Perfect Scoring Tutor With Over a Decade of Experience
MartyMurrayCoaching.com
Contact me at [email protected] for a free consultation.

User avatar
Master | Next Rank: 500 Posts
Posts: 410
Joined: Fri Mar 13, 2015 3:36 am
Location: Worldwide
Thanked: 120 times
Followed by:8 members
GMAT Score:770

by OptimusPrep » Thu May 19, 2016 3:46 am
eitijan wrote:Source: GMATPrep

Experts, please comment.
OA E


Gortland is narrowly self-sufficient in both grain and meat. Per capita consumption of meat has risen toward the world average, and it takes several pounds of grain to produce one pound of meat.

Therefore, since domestic grain production will not increase, Gortland will have to import either grain or meat or both.

We need to find the underlying assumption behind the conclusion that wither grain or meat has to be exported.
Possible assumptions can be:
1. Gortland will not start producing excess grain - we know this
2. People will not alter their food habits

Looking at the options, Option E resonates with our point 2.

Correct Option: E